Đến nội dung

Rias Gremory nội dung

Có 306 mục bởi Rias Gremory (Tìm giới hạn từ 10-06-2020)



Sắp theo                Sắp xếp  

#578127 Cơ bản về nguyên lý Đi-rích-lê

Đã gửi bởi Rias Gremory on 03-08-2015 - 13:10 trong Toán rời rạc

Đóng góp cho topic 1 bài diriclet

Có 9 điểm trên mặt phẳng, bất cứ điểm nào cũng là đỉnh của một tam giác mà cạnh được tô bởi màu xanh hoặc đỏ trong đó bao giờ cũng có cạnh màu đỏ. CMR tồn tại 1 tứ giác có các cạnh và đường chéo cùng màu đỏ

Trường hợp $1$ : Trong $9$ điểm tồn tại $1$ điểm $a$ là đầu mút của $4$ cạnh xanh.

Ta xét $4$ đỉnh kề với $a$ bởi các cạnh xanh. Khi đó $4$ đỉnh này sẽ lập thành $1$ tứ giác mà các cạnh và đường chéo được tô màu đỏ . 

Vì nếu trong tứ giác mà tồn tại $1$ cạnh màu xanh thì cạnh đó cùng với $2$ cạnh trong $4$ cạnh xanh kia sẽ lập nên $1$ tam giác không có cạnh màu đỏ.

Trái với giả thiết . Vậy tứ giác đó là tứ giác cần tìm

Trường hợp $2$ : Tất cả các đỉnh có số cạnh xanh xuất phát từ đỉnh đó không vượt quá $3$. Khi đó tồn tại $1$ đỉnh mà có số cạnh xanh xuất phát từ nó $\leq 2$ .

Vì nếu tất cả các đỉnh đều có số cạnh xanh là $3$ thì số cạnh xanh của đồ thị là $\frac{9.3}{2}=\frac{27}{2}$ (vô lý)

Gọi $a$ là đầu mút có nhiều nhất $2$ cạnh xanh , khi đó $a$ có ít nhất $6$ cạnh đỏ

Xét $6$ đỉnh kề với $a$ mà được nối với $a$ bởi cạnh màu đỏ, $6$ điểm này lập thành $1$ đồ thị đủ $6$ đỉnh mà các cạnh mà các cạnh được tô bởi $2$ màu xanh và đỏ.

Áp dụng định lí Ramsey : Nếu tô màu các cạnh của đồ thị đầy đủ $6$ đỉnh $K_6$ với $2$ màu xanh đỏ , thì luôn tồn tại $1$ đồ thị đầy đủ $3$ đỉnh $K_3$ là đồ thị con của đồ thị này và tất cả các cạnh của nó hoặc cùng màu đỏ hoặc cùng màu xanh

Theo định lí trên, luôn tồn tại $3$ đỉnh lập thành $1$ tam giác mà các cạnh được tô cùng một màu xanh hoặc đỏ

Nhưng tam giác này không thể là màu xanh vì nó không chứa cạnh màu đỏ , trái với giả thiết. Vậy tam giác đó chỉ có thể là màu đỏ

Khi đó $3$ đỉnh này cùng với đỉnh $a$ lập thành một tứ giác mà tất cả các cạnh và đường chéo cùng được tô màu đỏ . (ĐPCM)

Hình gửi kèm

  • Trường hợp 1.JPG
  • Trường hợp 2.JPG



#571471 MỘT SỐ PHƯƠNG PHÁP GIẢI TOÁN TỔ HỢP THCS

Đã gửi bởi Rias Gremory on 11-07-2015 - 17:45 trong Toán rời rạc

Bài 4: Giả sử $a_{1};a_{2};...;a_{11}$ là các số nguyên dương lớn hơn hoặc bằng $2$,đôi một khác nhau và thoả mãn 

$a_{1}+a_{2}+...+a_{11}=407$.Cm rằng không tồn tại số nguyên dương $n$ sao cho tổng các số dư của các phép chia $n$ cho $22$ số $a_{1};a_{2};...;a_{11}$;$4a_{1};4a_{2};...;4a_{11}$ bằng $2012$

$n\equiv 4a-1$ ( mod $4a$) $\Rightarrow n\equiv a-1$ (mod $a$ )  (*)

Giả sử tồn tại $n$ thõa mãn yêu cầu bài toán 

Nếu như mọi phép chia đều thiếu $1$

$\Rightarrow$ Tổng số dư của các phép chia $n$ cho $a_1,a_2,...,a_{11},4a_1,4a_2,...,4a_{11}$ là $407+4.407-11-11=2013$

Do tổng các số dư $=2012$ nên phải có $21$ phép chia đều thiếu $1$ và$1$ phép chia thiếu $2$

Nếu phép chia $n$ cho $a_1$ thiếu $2$ mà $n$ chia $4a_1$ thiếu $1$ sẽ mâu thuẫn với (*)

$\Rightarrow$ Không tồn tại $n$ thõa mãn YCBT




#581173 Ảnh thành viên

Đã gửi bởi Rias Gremory on 12-08-2015 - 23:38 trong Góc giao lưu

SAO CÁI ANH ÁO TRẮNG QUẦN ĐỎ CHÓI THÉ KIA MÀ KHÔNG THẤY MẶT Ạ  :closedeyes:

Kệ anh í nờ  :D , anh chưa nhận xét , tò mò cái anh đó làm chi trời :D 




#581175 Ảnh thành viên

Đã gửi bởi Rias Gremory on 12-08-2015 - 23:46 trong Góc giao lưu

TẠI EM NGHĨ ANH Ý ĐẸP TRAI HƠN  :closedeyes:

Chuẩn rồi :D , đẹp trai hơn , nhà giàu hơn , nhiều bồ hơn , thông minh hơn , được cái không xấu trai hơn anh :D




#591073 Ảnh thành viên

Đã gửi bởi Rias Gremory on 27-09-2015 - 08:24 trong Góc giao lưu

Nhìn qua, anh hy vọng nick này là người trước, ai ngờ là người đứng sau. Xuống đất  :wacko:

Hy vọng càng nhiều thất vọng nhiều hơn thôi a :D 




#581167 Ảnh thành viên

Đã gửi bởi Rias Gremory on 12-08-2015 - 23:23 trong Góc giao lưu

Có tấm gọi là :)

Ném đá thoải mái mấy chế :D

 

11665613_1623507787890814_61999272611328




#570656 Chuyên đề : Làm mạnh BĐT CôSy

Đã gửi bởi Rias Gremory on 08-07-2015 - 23:37 trong Bất đẳng thức và cực trị

 Áp dụng BĐT AM-GM ta có :

 $\frac{a^3}{b^2}+a\geq \frac{2a^2}{b}\Rightarrow \frac{a^3}{b^2}+\frac{b^3}{c^2}+\frac{c^3}{a^2}\geq 2-(a+b+c)$

 Vì vậy ta chỉ cần chứng minh 

      $2-(a+b+c)\geq 1+\frac{a^2}{2b^2}.(a-b)^2+\frac{b^2}{2c^2}.(b-c)^2+\frac{c^2}{2a^2}.(c-a)^2$

      $\Leftrightarrow \frac{a^2}{b}+\frac{b^2}{c}+\frac{c^2}{a}\geq a+b+c+\frac{a^2}{2b^2}.(a-b)^2+\frac{b^2}{2c^2}.(b-c)^2+\frac{c^2}{2a^2}.(c-a)^2$

 Theo BĐT (3) thì ta cần chỉ ra $\frac{a^2}{2b},\frac{b^2}{2c},\frac{c^2}{2a}\in (0;1]$

 Mà ta lại có : $\frac{a^2}{b}+\frac{b^2}{c}+\frac{c^2}{a}=1\Rightarrow \frac{a^2}{b}<1<2\Rightarrow \frac{a^2}{2b}<1$

 Từ đó có điều cần chứng minh

Bài này còn cách giải khác là áp dụng BĐT $(10)$ 

Với $\alpha =\frac{3b}{4(a+b)}.\frac{a^2}{b}; \beta =\frac{3c}{4(b+c)}.\frac{b^2}{c}; \gamma =\frac{3a}{4(c+a)}.\frac{c^2}{a}$

Ngày mai sẽ post tiếp bài tập nhé :D :D :D 




#570491 Chuyên đề : Làm mạnh BĐT CôSy

Đã gửi bởi Rias Gremory on 08-07-2015 - 10:48 trong Bất đẳng thức và cực trị

Em thấy bài tập 1 đâu nhất thiết phải $0<a,b,c<1$

Phải có thêm điều kiện thì mới chọn được bộ $\alpha ,\beta ,\gamma$ nhé :D

 

bt2:(*)$\Leftrightarrow 2(a^{2}+b^{2}+c^{2})-(ab^{2}+ac^{2}+bc^{2}+ca^{2}+ba^{2}+cb^{2})\geq 2(ab+bc+ca)-abc(\frac{a}{b}+\frac{b}{a}+\frac{a}{c}+\frac{c}{a}+\frac{c}{b}+\frac{b}{c})=2-abc(2+2+2)=2-6abc(đpcm)$

Dấu''='' xảy ra $\Leftrightarrow a=b=c$

 

BT2

ap dung bdt cauchy

(*)$\geq (1-a)2bc+(1-b)2ca+(1-c)2ab =2-6abc$(đpcm)

Dấu "=" xảy ra $\Leftrightarrow a=b=c$

 

Lâu rồi không ghé vào TOPic , thấy người LIKE khá nhiều , đứng TOP 9 của 4RUM , nên mình post tiếp một số bài tập :D , ai quan tâm thì vào làm nhé !!

BT$1$ : Cho $a,b,c> 0,a+b+c=1$ . Chứng minh rằng :

$6(ab+bc+ca)+a(a-b)^2+b(b-c)^2+c(c-a)^2\leq 2$

BT$2$ : Với $a,b,c> 0,ab+bc+ca=1$ . Chứng minh 

$(1-a)(b^2+c^2)+(1-b)(c^2+a^2)+(1-c)(a^2+b^2)\geq 2-6abc$

 

Chủ yếu mình post bài tập dễ cho mấy bạn làm quen với Chuyên đề này :D :D :D

Cách giải

Bài $1$ : 
Vì $a,b,c >0$ và $a+b+c=1$ suy ra $0< a,b,c< 1$
Sử dụng  BĐT (2) với $\alpha =a,\beta =b,\gamma =c$ ta có : 
$a^2+b^2+c^2\geq ab+bc+ca+\frac{a(a-b)^2}{2}+\frac{b(b-c)^2}{2}+\frac{c(c-a)^2}{2}$
$\Leftrightarrow 1=(a+b+c)^2\geq 3(ab+bc+ca)+\frac{a(a-b)^2}{2}+\frac{b(b-c)^2}{2}+\frac{c(c-a)^2}{2}$
$\Leftrightarrow 2\geq 6(ab+bc+ca)+a(a-b)^2+b(b-c)^2+c(c-a)^2 (dpcm)$
Bài $2$ : 
Từ ĐK suy ra $0< a,b,c< 1$
Sử dụng  BĐT (2) với $\alpha =a,\beta =b,\gamma =c$ ta có : 
$a^2+b^2+c^2\geq ab+bc+ca+\frac{a(a-b)^2}{2}+\frac{b(b-c)^2}{2}+\frac{c(c-a)^2}{2}$
$\Leftrightarrow 2(a^2+b^2+c^2)\geq 2+a(b^2+c^2-2bc)+b(c^2+a^2-2ca)+c(a^2+b^2-2ab)$
$\Leftrightarrow (1-a)(b^2+c^2)+(1-b)(c^2+a^2)+(1-c)(a^2+b^2)\geq 2-6abc$ (ĐPCM)



#589669 Chuyên đề : Làm mạnh BĐT CôSy

Đã gửi bởi Rias Gremory on 18-09-2015 - 20:25 trong Bất đẳng thức và cực trị

 @PhamHungCxHT:Bổ sung phần cm cho bất đẳng thức $(4)$ với cái dòng đầu cm $(5)$ chưa được cm !

À mà anh không viết tiếp chuyên đề nữa ạ  :(

Từ BĐT (4) suy ra $\frac{a^{m+n}+b^{m+n}}{2}\geq (\frac{a+b}{2})^{m+n}+\frac{\alpha }{4}(a^m-b^m)(a^n-b^n)$




#589960 Chuyên đề : Làm mạnh BĐT CôSy

Đã gửi bởi Rias Gremory on 20-09-2015 - 11:51 trong Bất đẳng thức và cực trị

BĐT $11$

Với $a,b,c>0, 0<\alpha ,\beta ,\gamma <1$ . $m,n$ là các số tự nhiên . Chứng minh : 

$\frac{a^{m+n}+b^{m+n}+c^{m+n}}{3}\geq (\frac{a+b+c}{3})^{m+n}+\frac{\alpha }{9}(a^m-b^m)(a^n-b^n)+\frac{\beta }{9}(a^m-c^m)(a^n-c^n)+\frac{\gamma }{9}(b^m-c^m)(b^n-c^n)$

 

Lời giải : 

$\frac{a^m+b^m+c^m}{3}\geq (\frac{a+b+c}{3})^m$

$\frac{a^n+b^n+c^n}{3}\geq (\frac{a+b+c}{3})^n$

Nên bất đẳng thức này suy trực tiếp từ BĐT $8$




#589961 Chuyên đề : Làm mạnh BĐT CôSy

Đã gửi bởi Rias Gremory on 20-09-2015 - 11:53 trong Bất đẳng thức và cực trị

Bài tập nhé : 

$15$ , Với $a,b,c$ thõa mãn $a\geq 2b\geq 4c>0$ . Chứng minh : 

$a^2+3b^2+5c^2\geq 2(ab+bc+ca)+\frac{1}{3}(b^3+c^3)+\frac{c^3}{b}$




#570495 Chuyên đề : Làm mạnh BĐT CôSy

Đã gửi bởi Rias Gremory on 08-07-2015 - 10:56 trong Bất đẳng thức và cực trị

Khuấy động TOPIC tiếp nào :D . Chắc chuyên đề hơi khó nhỉ :D

Bài tập $9$ 

Với $a,b,c >0$ . Chứng minh :

$\frac{a^3}{b}+\frac{b^3}{c}+\frac{c^3}{a}\geq ab+bc+ca+\frac{(a-b)^2}{2}+\frac{(b-c)^2}{2}+\frac{(c-a)^2}{2}$

Gợi ý : Dùng BĐT (7) :D , Bộ số các bạn tự tìm nha




#589668 Chuyên đề : Làm mạnh BĐT CôSy

Đã gửi bởi Rias Gremory on 18-09-2015 - 20:23 trong Bất đẳng thức và cực trị

 @PhamHungCxHT:Bổ sung phần cm cho bất đẳng thức $(4)$ với cái dòng đầu cm $(5)$ chưa được cm !

À mà anh không viết tiếp chuyên đề nữa ạ  :(

Chuyên đề này khá lạ , nên ít người tham gia thảo luận lắm em à :D , anh cũng chán lắm chứ !! Không có ai thảo luận , chỉ đọc thấy vui vui rồi like cái xong lượn đi :(




#570830 Chuyên đề : Làm mạnh BĐT CôSy

Đã gửi bởi Rias Gremory on 09-07-2015 - 21:09 trong Bất đẳng thức và cực trị

Em có 1 thắc mắc ạ.Nếu như theo AM-GM bộ 3 số thì ta sẽ có $\frac{a}{b}+\frac{b}{c}+\frac{c}{a}\geq 3$ sẽ không bé hơn $1$

Dẫn đến gt sai phải không ạ?? :(

Ừ . giả thiết sai rồi , haizz . Bất cẩn quá không để ý , để kiếm bài tập post lên tiếp nào :)) 

Em làm xong rồi ms nhìn lại . hay ha :D




#544704 Chuyên đề : Làm mạnh BĐT CôSy

Đã gửi bởi Rias Gremory on 17-02-2015 - 20:42 trong Bất đẳng thức và cực trị

Lâu rồi không ghé vào TOPic , thấy người LIKE khá nhiều , đứng TOP 9 của 4RUM , nên mình post tiếp một số bài tập :D , ai quan tâm thì vào làm nhé !!

BT$7$ : Cho $a,b,c> 0,a+b+c=1$ . Chứng minh rằng :

$6(ab+bc+ca)+a(a-b)^2+b(b-c)^2+c(c-a)^2\leq 2$

BT$8$ : Với $a,b,c> 0,ab+bc+ca=1$ . Chứng minh 

$(1-a)(b^2+c^2)+(1-b)(c^2+a^2)+(1-c)(a^2+b^2)\geq 2-6abc$




#570655 Chuyên đề : Làm mạnh BĐT CôSy

Đã gửi bởi Rias Gremory on 08-07-2015 - 23:33 trong Bất đẳng thức và cực trị

Cung cấp các BĐT tiếp nhé :D

BĐT $(8)$

Với $a,b,c>0;0<\alpha ,\beta ,\gamma <1$ , $m,n$ là các số tự nhiên . 

Ta có : $a^{m+n}+b^{m+n}+c^{m+n}\geq \frac{1}{3}(a^m+b^m+c^m)(a^n+b^n+c^n)+\frac{\alpha }{3}(a^m-b^m)(a^n-b^n)+\frac{\beta  }{3}(a^m-c^m)(a^n-c^n)+\frac{\gamma  }{3}(b^m-c^m)(b^n-c^n)$

Chứng minh

$BDT(8)\Leftrightarrow (1-\alpha )(a^m-b^m)(a^n-b^n)+(1-\beta )(a^m-c^m)(a^n-c^n)+(1-\gamma )(b^m-c^m)(b^n-c^n)$

 

BĐT $(9)$ 

Với $a,b,c>0;0<\alpha ,\beta ,\gamma <1$

Ta có :  $\frac{a^2}{b^2}+\frac{b^2}{c^2}+\frac{c^2}{a^2}\geq \frac{a}{b}+\frac{b}{c}+\frac{c}{a}+\frac{\alpha }{b^2}(a-b)^2+\frac{\beta }{c^2}(b-c)^2+\frac{\gamma }{a^2}(c-a)^2$

Chứng minh 

Từ BĐT(1) :  $a^2+b^2\geq 2ab+\alpha (a-b)^2$ ta suy ra : 

$\frac{a^2}{b^2}+1\geq \frac{2a}{b}+\frac{\alpha }{b^2}(a-b)^2$

Tương tự $\frac{b^2}{c^2}+1\geq \frac{2b}{c}+\frac{\beta }{c^2}(b-c)^2$

$\frac{c^2}{a^2}+1\geq \frac{2c}{a}+\frac{\gamma }{a^2}(c-a)^2$

$\frac{a}{b}+\frac{b}{c}+\frac{c}{a}\geq 3$

Cộng vế với vế ta được ĐPCM

 

BĐT $(10)$

Với $a,b,c>0;0<\alpha ,\beta ,\gamma <1$

$\frac{a^3}{b^2}+\frac{b^3}{c^2}+\frac{c^3}{a^2}\geq \frac{a^2}{b}+\frac{b^2}{c}+\frac{c^2}{a}+\frac{2\alpha }{3b^2}(a^2-b^2)(a-b)+\frac{2\beta }{3c^2}(b^2-c^2)(b-c)+\frac{2\gamma }{3a^2}(c^2-a^2)(c-a)$

Chứng minh

Từ BĐT(5) : $a^3+b^3\geq ab(a+b)+\frac{2\alpha }{3}(a^2-b^2)(a-b)$ suy ra

$\frac{a^3}{b^2}+b\geq \frac{a^2}{b}+a+\frac{2\alpha }{3b^2}(a^2-b^2)(a-b)$

Tương tự 

$\frac{b^3}{c^2}+c\geq \frac{b^2}{c}+b+\frac{2\beta }{3c^2}(b^2-c^2)(b-c)$

$\frac{c^3}{a^2}+a\geq \frac{c^2}{a}+c+\frac{2\gamma }{3a^2}(c^2-a^2)(c-a)$

Cộng vế với vế ta được ĐPCM




#570690 Chuyên đề : Làm mạnh BĐT CôSy

Đã gửi bởi Rias Gremory on 09-07-2015 - 10:31 trong Bất đẳng thức và cực trị

Tiếp tục nào 

Bài $11$ :

Với $a,b,c>0;\frac{a}{b}+\frac{b}{c}+\frac{c}{a}=1$. Chứng minh

$\frac{a^2}{b^2}+\frac{b^2}{c^2}+\frac{c^2}{a^2}\geq 1+\frac{(a-b)^2}{c^2}+\frac{(b-c)^2}{a^2}+\frac{(c-a)^2}{b^2}$

Bài $12$ :

Với $a,b,c>0$. Chứng minh

$\frac{a^3}{b^2}+\frac{b^3}{c^2}+\frac{c^3}{a^2}\geq \frac{a^2}{b}+\frac{b^2}{c}+\frac{c^2}{a}+\frac{(a-b)^2}{2b}+\frac{(b-c)^2}{2c}+\frac{(c-a)^2}{2a}$



#570638 Chuyên đề : Làm mạnh BĐT CôSy

Đã gửi bởi Rias Gremory on 08-07-2015 - 22:39 trong Bất đẳng thức và cực trị

Giải thế này đúng không ấy nhỉ  :closedeyes:

Đặt $\alpha =\frac{3b}{4(a+b)};\beta =\frac{3c}{4(b+c)};\gamma =\frac{3a}{4(a+c)}$

Dễ thấy $\alpha ;\beta ;\gamma > 0$ 

Ta có $1-\alpha =1-\frac{3b}{4(a+b)}=\frac{4a+b}{4(a+b)}> 0\rightarrow \alpha < 1$

Tương tự ta kết luận $0< \alpha ;\beta ;\gamma < 1$

Thay vào bất đẳng thức trên ta có bất đẳng thức $(7)$ 

$\bullet$ $\frac{a^{3}}{b}+\frac{b^{3}}{c}+\frac{c^{3}}{a}\geq ab+bc+ca+\frac{2\alpha }{3b}(a^{2}-b^{2})(a-b)+\frac{2\beta }{3c}(b^{2}-c^{2})(b-c)+\frac{2\gamma }{3a}(c^{2}-a^{2})(c-a)$

Mà bất đẳng thức này anh PhamhungCxHt đã cm nên khỏi cần cm lại  >:)

Xong bài toán  :icon6:

Đúng rồi đó :D , chủ yếu là tìm đc bộ số thôi :D

Bài $10$ 

Với $a,b,c >0$ và $\frac{a^2}{b}+\frac{b^2}{c}+\frac{c^2}{a}=1$. Chứng minh rằng :

$\frac{a^3}{b^2}+\frac{b^3}{c^2}+\frac{c^3}{a^2}\geq 1+\frac{a^2}{2b^2}.(a-b)^2+\frac{b^2}{2c^2}.(b-c)^2+\frac{c^2}{2a^2}.(c-a)^2$




#575944 Chuyên đề : Làm mạnh BĐT CôSy

Đã gửi bởi Rias Gremory on 27-07-2015 - 20:06 trong Bất đẳng thức và cực trị

Tiếp tục chuyên đề 

$13$,

Cho $a,b,c>0$ . CHứng minh :

$\frac{1}{a}+\frac{1}{b}+\frac{1}{c}\geq \frac{2}{a+b}+\frac{3}{b+c}+\frac{1}{c+a}+\frac{1}{\sqrt{ab}}+\frac{1}{2\sqrt{bc}}+\frac{3}{2\sqrt{ca}}$

$14$.

Với $a,b,c >0 ; a+b+c=1$ . Chứng minh rằng 

$\frac{1}{a}+\frac{1}{b}+\frac{1}{c}\geq 2(\frac{1-2\sqrt{bc}}{1-a}+\frac{1-2\sqrt{ca}}{1-b}+\frac{1-2\sqrt{ab}}{1-c})$




#628901 Tuyển tập các đề thi thử đại học năm 2016

Đã gửi bởi Rias Gremory on 22-04-2016 - 13:29 trong Thi TS ĐH

Câu $1$,

Khảo sát sự biến thiên và vẽ đồ thị hàm số $y=-x^3+3x+1$.

Câu $2$,

Viết phương trình tiếp tuyến của đồ thị hàm số $y=\frac{x+1}{x-2}$ tại điểm có hoành độ bằng $1$.

Câu $3$,

$a,$ Cho số phức $z$ thỏa mãn $z(2+i)+\overline{z}=5+3i$. Tính môđun của số phức $z$.

$b,$ Giải phương trình : $log_{2}(3x-1)+log_{2}(x+3)-3=0$

Câu $4$,

Tính tích phân :$\int_{1}^{2}x(1+ln2x)dx$
Câu $5$, 

Trong không gian với hệ tọa độ $Oxyz$, cho mặt phẳng $(P):2x-y+2z+2=0$ và điểm $M(1;2;3)$. Viết phương trình đường thẳng đi qua $M$, vuông góc với mặt phẳng $(P)$ và tìm tọa độ điểm $N$ đối xứng với điểm $M$ qua mặt phẳng $(P)$.

Câu $6$,

$a,$ Giải phương trình : $cos2x=5cosx-3$

$b,$ Trong dịp $26/3$, Đoàn trường của một trường Trung học phổ thông chọn ngẫu nhiên $6$ đoàn viên xuất sắc thuộc ba khối $10,11$ và $12$, mỗi khối $2$ đoàn viên xuất sắc để tuyên dương. Biết khối $10$ có $4$ đoàn viên xuất sắc trong đó có hai nam và hai nữ, khối $11$ có $5$ đoàn viên xuất sắc trong đó có hai nam và ba nữ, khối $12$ có $6$ đoàn viên xuất sắc trong đó có ba nam và ba nữ. Tính xác suất để $6$ đoàn viên xuất sắc được chọn có cả nam và nữ.

Câu $7$,

Cho hình chóp $S.ABCD$ đáy hình chữ nhật có cạnh $AB=a,AD=2a$. Gọi $O$ là giao điểm của hai đường thẳng $AC$ và $BD$, $G$ là trọng tâm tam giác $SAD$. Biết $SO$ vuông góc với mặt phẳng $(ABCD)$, góc giữa đường thẳng $SC$ và mặt phẳng $(ABCD)$ bằng $60^o$. Tính theo $a$ thể tích của khối chóp $S.ABCD$ và khoảng cách từ điểm $G$ đến mặt phẳng $SCD$/

Câu $8$,

Trong mặt phẳng với hệ tọa độ $Oxy$, cho tam giác $ABC$ cân tại $C$. Các điểm $M,N$ lần lượt là chân đường cao hạ từ $A$ và $C$ của tam giác $ABC$. Trên tia đối của tia $AM$ lấy điểm $E$ sao cho $AE=AC$. Biết tam giác $ABC$ có diện tích bằng $8$, đường thẳng CN có phương trình $y-1=0$, điểm $E(-1;7)$, điểm $C$ có hoành độ dương và điểm $A$ có tọa độ là các số nguyên. Tìm tọa độ các đỉnh của tam giác $ABC$.

Câu $9$,

Giải phương trình : $(2x^2-2x+1)(2x-1)+(8x^2-8x+1)\sqrt{-x^2+x}=0$

Câu $10$,

Cho các số thực dương $x,y,z$ thỏa mãn : $\frac{1}{x}+\frac{1}{y}+\frac{1}{z}=\frac{16}{x+y+z}$. Tìm giá trị lớn nhất của biểu thức :

$P=\frac{(x-y)(y-z)(z-x)}{xyz}$




#567796 Chuẩn hóa Bất đẳng thức

Đã gửi bởi Rias Gremory on 24-06-2015 - 11:05 trong Tài liệu, chuyên đề, phương pháp về Bất đẳng thức

Như tiêu đề , em muốn xin một số kỹ thuật chuẩn hóa bất đẳng thức và khái niệm , phần này em không hiểu lắm !

 

Một biểu thức $P(a,b,c,...,u,v)$ gọi là thuần nhất bậc $k$ nếu và chỉ nếu với mọi số thực $t$ khác $0$, ta đều có
\[{t^k}P\left( {a,b,c,...,u,v} \right) = P\left( {ta,tb,tc,...,tu,tv} \right)\]
Một bất đẳng thức gọi là thuần nhất nếu cả hai vế của nó đều là những biểu thức thuần nhất. Xét một bất đẳng thức thuần nhất bậc $k$ 3 biến (thực ra bao nhiêu biến không quan trọng, ở đây cho gọn xin phép chỉ xét 3 biến):
\[A\left( {a,b,c} \right) \ge B\left( {a,b,c} \right)\]
Giả sử bộ số $(a,b,c)$ thỏa mãn bất đẳng thức trên. Đặt $S=a+b+c$ (hoặc một biểu thức nào đó tùy ý). Từ đó suy ra $\dfrac{a}{S} + \dfrac{b}{S} + \dfrac{c}{S} = 1$. Do tính thuần nhất của bất đẳng thức trên nên ta có biến đổi sau:

\[\begin{array}{l}
A\left( {a,b,c} \right) \ge B\left( {a,b,c} \right) \\
\Leftrightarrow \dfrac{1}{{{S^k}}}A\left( {a,b,c} \right) \ge \dfrac{1}{{{S^k}}}B\left( {a,b,c} \right) \\
\Leftrightarrow A\left( {\dfrac{a}{S},\dfrac{b}{S},\dfrac{c}{S}} \right) \ge B\left( {\dfrac{a}{S},\dfrac{b}{S},\dfrac{c}{S}} \right) \\
\end{array}\]
Tức là bộ số $\left( {\dfrac{a}{S},\dfrac{b}{S},\dfrac{c}{S}} \right)$ cũng thỏa mãn bất đẳng thức đang xét. Vậy ta có thể chỉ chứng minh bất đẳng thức với bộ số $\left( {\dfrac{a}{S},\dfrac{b}{S},\dfrac{c}{S}} \right)$ là đủ. Mà đối với bộ số mới này, chúng có tổng là $1$. Để cho gọn, người ta ghi "chuẩn hóa: $a+b+c=1$".

 

Bạn có thể đọc thêm ở đây




#643132 Kì thi THPTQG 2016 - môn Toán

Đã gửi bởi Rias Gremory on 01-07-2016 - 12:29 trong Thi TS ĐH

mình nghĩ phải xét hai trường hợp chứ nhỉ nếu điểm E nằm giữa N và B thì $\widehat{PEN}\leq 90^0$ mà trường hợp này cũng tính ra được là A là điểm chính giữa cung BD

Bạn vẽ trường hợp nào cũng vô lý hết , và suy ra A nằm giữa cung BD , từ đó nên P trùng N , suy ra vô lý




#643127 Kì thi THPTQG 2016 - môn Toán

Đã gửi bởi Rias Gremory on 01-07-2016 - 12:03 trong Thi TS ĐH

Thì có gì đâu em? Nghiệm hình vẫn đúng mà!

Ta có : $\widehat{NPE}=90^0\Rightarrow \widehat{PNE}+\widehat{NEP}=90^0\Rightarrow \widehat{PEN}\leq 90^0$ 

Mà $\widehat{PEN}=\widehat{DEA}=\widehat{EAN}+\widehat{ANE}=\widehat{EAN}+90^0\Rightarrow \widehat{PEN}\geq 90^0$

Hình gửi kèm

  • aa.jpg



#643117 Kì thi THPTQG 2016 - môn Toán

Đã gửi bởi Rias Gremory on 01-07-2016 - 11:37 trong Thi TS ĐH

Có ai xem giùm câu $8$ được không , giả thiết cho MN vuông góc AC , quá vô lý ở cái hình . 




#643110 Kì thi THPTQG 2016 - môn Toán

Đã gửi bởi Rias Gremory on 01-07-2016 - 11:27 trong Thi TS ĐH

Có sự vô lý ở câu $8$ khi đề đã cho MN vuông góc AC .